\documentclass[11pt]{scrartcl} \usepackage[headheight=1pt, includeheadfoot, headsep=1cm, top=1.5cm, bottom=1.3cm, left=2cm, right=2cm]{geometry} \usepackage{mathtools} \usepackage{amsmath,amsthm, amscd, amssymb, amsfonts} \usepackage[english]{babel} \usepackage{fancyhdr} \usepackage{enumitem} \setlist{itemsep=0.5em} \usepackage{xcolor} \definecolor{forestgreen}{rgb}{0.13, 0.55, 0.13} \usepackage[colorlinks = true, linkcolor = forestgreen, urlcolor = forestgreen, citecolor = forestgreen, anchorcolor = forestgreen]{hyperref} \usepackage{multicol} \usepackage{multienum} \usepackage{euler} \usepackage[OT1]{eulervm} \renewcommand{\rmdefault}{pplx} \usepackage{tikz} \usepackage{mathabx} %\usepackage{lastpage} %% header, footer definitions \def\course{MATH 314} \def\courselink{https://www.gsanmarco.com/graph-theory} \def\assignment{HW \#1} \def\assignlink{hw1.tex} \def\due{Due on Jan 24} \pagestyle{fancy} \fancyhead[L]{\course} \fancyhead[C]{\assignment} \fancyhead[R]{\due} \renewcommand{\footrulewidth}{0.4pt} \fancyfoot{} %\fancyfoot[R]{\fontsize{8}{10} \selectfont Page \thepage~of \pageref{LastPage}} \fancyfoot[L]{\fontsize{10}{12} \selectfont Source file available at \href{\courselink}{\textbf{\courselink}}} %% environments % theorem style \newtheorem{theorem}{Theorem} \newtheorem{lemma}[theorem]{Lemma} \newtheorem{corollary}[theorem]{Corollary} \newtheorem{claim}[theorem]{Claim} \newtheorem{defn}[theorem]{Definition} % definition style \theoremstyle{definition} \newtheorem{problem}{Problem} % custom \newenvironment{solution}{\noindent\textbf{Solution.}}{\qed} %% custom commands \newcommand*{\abs}[1]{\lvert #1\rvert} % absolute values, cardinality \newcommand*{\eps}{\varepsilon} % prettier epsilon \newcommand*{\R}{\mathbb{R}} % real numbers \newcommand*{\C}{\mathbb{C}} % complex numbers \newcommand*{\Q}{\mathbb{Q}} % rational numbers \newcommand*{\Z}{\mathbb{Z}} % integers \newcommand*{\N}{\mathbb{N}} % natural numbers \newcommand*{\F}{\mathbb{F}} % field \newcommand*{\family}{\mathcal{F}} % family \newcommand*{\GL}{\mathbf{GL}} % general linear group \newcommand*{\what}[1]{\widehat{#1}} % wider hat \newcommand*{\wbar}[1]{\overline{#1}} % wider bar \newcommand*{\mcal}[1]{\mathcal{#1}} % mathcal \newcommand*{\mbf}[1]{\mathbf{#1}} % mathbf \newcommand*{\mbb}[1]{\mathbb{#1}} % mathbb % operators \let\Pr\relax\DeclareMathOperator*{\Pr}{\mathbf{Pr}} % probability \DeclareMathOperator*{\E}{\mathbb{E}} % expectation \DeclareMathOperator*{\Var}{\mathbf{Var}} % variance \DeclareMathOperator*{\Span}{span} % span \DeclareMathOperator{\tr}{tr} % trace \DeclareMathOperator{\rank}{rank} % rank \DeclareMathOperator{\supp}{supp} % support \DeclareMathOperator{\diam}{diam} \begin{document} \noindent Unless explicitly requested by a problem, do not include sketches as part of your proof. You are free to use the result from any problem on this (or previous) assignment as a part of your solution to a different problem even if you have not solved the former problem. \vskip10pt \begin{problem}[2 pts] Let $S$ be a finite set of integers and let $G$ be the graph on vertex set $S$ where for any $x,y\in S$, $xy\in E(G)$ if and only if $x+y$ is odd. \begin{enumerate} \item Prove that $G$ is a bipartite graph for any such $S$. \item If $S=\{1, 2, \dots, 100\}$, what is $\abs{E(G)}$? Justify your answer. \end{enumerate} \end{problem} \begin{problem}[2 pts] Suppose that $(u=v_0,v_1,\dots,v_k=v)$ is a $u$--$v$ geodesic. Prove that $d(u,v_i)=i$ for all $i\in\{0,\dots,k\}$. \end{problem} \begin{problem}[2 pts]\label{abpath} Let $G$ be a graph. For two non-empty subsets $A,B\subseteq V(G)$, an $A$--$B$ path is a path in $G$ which connects some vertex of $A$ to some vertex of $B$. Prove that if $P$ is a minim\textbf{al} $A$--$B$ path, then $P$ contains exactly one vertex from $A$ and contains exactly one vertex from $B$. \end{problem} \begin{problem}[2 pts] Let $G$ be a connected graph. Prove that any two maxim\textbf{um} paths in $G$ must share some vertex. \end{problem} \begin{problem}[2 pts] Prove that a graph $G$ is bipartite if and only if every subgraph $H$ of $G$ has an independent set consisting of at least half of $V(H)$. (recall that an independent set is a set of vertices which induce no edges) \end{problem} \end{document}